解析力学/ラグランジアン のバックアップ差分(No.2)

更新


  • 追加された行はこの色です。
  • 削除された行はこの色です。
&katex();
* ラグランジアン [#oa193382]

* 目次 [#z557608a]

#contents

* ラグランジアンの定義 [#oa193382]

力学系の運動エネルギーを $T$、ポテンシャルエネルギーを $U$ とするとき、
ラグランジアン $L$ を、

$$
L=T-U
$$

として定義する。

* 力学系の座標と速度 [#a3b96963]

その力学系を記述するための座標が $q_1,q_2,\dots,q_n$ で表されるとしよう。

>3次元中での質点1個の運動なら $n=3$ で、$q_1=x,q_2=y,q_3=z$ と考えればいい。

>3次元中での質点2個の運動なら $n=6$ で、$q_1=x_1,q_2=y_1,q_3=z_1,q_4=x_2,q_5=y_2,q_6=z_2$ と考えればいい。

>質点1が原点から距離 $r$ の球面上を動くなら、球座標を使って $n=2$ として $q_1=\theta,q_2=\phi$ と取れば良い。

また、それらの座標の時間微分を $\dot q_k=\frac{dq_k}{dt}$、 $\ddot q_k=\frac{d^2q_k}{dt^2}$ などと表すことにする。

一般に、ラグランジアンは $q_1,q_2,\dots,q_n$ および $\dot q_1,\dot q_2,\dots,\dot q_n$ の関数として表せる。($\ddot q_k$ などは含まない)

* 最小作用の原理 [#maf140c9]

時刻 $t_1$ の座標値 $q_k(t_1)$  と、時刻 $t_2$ の座標値 $q_k(t_2)$ が与えられたとすると、
その間での系の運動 $q_k(t)$ はラグランジアンの積分値として与えられる「作用」と呼ばれる値
その間での系の運動 $q_k(t)$ は「作用」と呼ばれる値

$$
S=\int_{t_1}^{t_2}L\,dt
S=\int_{t_1}^{t_2}L\ dt
$$

を最小にするようなものとなる。
を最小にするようなものとなる(ラグランジアンの積分値として与えられる)。

何を言っているかというと、各座標の最初と最後の値が決まってもその間で座標が時間と共にどのように変化するかはいろんな取り方がある。しかし、そのうち実際に実現する運動は $S$ が最小となるようなものになる、と言っている。
何を言っているかというと、各座標の最初と最後の値が決まってもその間で座標が時間と共にどのように変化するかはいろんな可能性が考えられる。しかし、そのうち実際に起きる運動は $S$ が最小とするものになる、と言っている。(一般に運動方程式は時間に対する2階の微分方程式であるから、1つの座標につき2つの自由パラメータを持つが、始点と終点という2つのパラメータを指定すると運動は1つに定まる(初期座標と初期速度を与えるのと同じ))

え、そんな話は聞いたことがないんだけど、と思うだろうけど実際そうなので仕方がない。
上記の話とニュートン方程式が成り立つ話とが矛盾しないことを以下で見る。
え、そんな話は聞いたことがないんだけど、と思うだろうけど実際こういう原理が存在するのだから仕方がないと諦めよう。上記の話とこれまで学んできたニュートン方程式が成り立つ話とが矛盾しないことを以下で見る。

* ラグランジュの運動方程式 [#x337268d]

「作用を最小化する」という条件を微分方程式の形で書いたのがラグランジュの運動方程式だ。

ある座標の軌跡 $q_k(t)$ を少しだけ変化させて $q_k(t)+\Delta q_k(t)$ としたとする。$q_k(t)$ が最小作用を与えるという意味は、どのように $\Delta q_k(t)$ を選んだとしても $S$ は元の値より増加する、ということと同値である。ただし始点と終点は決まっているので $\Delta q_k(t_1)=\Delta q_k(t_2)=0$ とする。
ある座標の軌跡 $q_k(t)$ を少しだけ変化させて $q_k(t)+\Delta q_k(t)$ としたとする。$q_k(t)$ が最小作用を与えるという意味は、どのように $\Delta q_k(t)$ を選んだとしても $S$ は元の値より増加する、ということと同義である。ただし始点と終点は決まっているので $\Delta q_k(t_1)=\Delta q_k(t_2)=0$ とする。

** 汎関数微分 [#u34af725]

$S$ は関数 $q_k(t)$ を与えると値が決まるという意味で「汎関数」と呼ばれ、これを強調する際には

$$S=S\big[q_k(t)\big]$$

などと書かれる。$S$ の $q_k$ による「汎関数微分 $\frac{\delta S}{\delta q_k}$」は、

$$
\begin{aligned}
\Delta S&=S\big[q_k(t)+\Delta q_k(t)\big]-S\big[q_k(t)\big]\\
&\sim\int_{t_1}^{t_2}\frac{\delta S}{\delta q_k}\Delta q_k(t)\,dt
\end{aligned}
$$

として定義される。

普通の関数 $f(x)$ の微分が

$$
\Delta f=f(x+\Delta x)-f(x)\sim\frac{df}{dx}\Delta x
\begin{aligned}
\Delta f&=f(x+\Delta x)-f(x)\\
&\sim\frac{df}{dx}\Delta x
\end{aligned}
$$

であるのと比べると、汎関数微分の定義になぜ積分が現れるのか疑問に思うかもしれない。この疑問は、多変数関数 $f(x_1,x_2,\dots,x_n)$ の全微分が、
であるのと比べると、汎関数微分の定義になぜ積分が現れるのか疑問に思うかもしれない。この疑問は、多変数関数 $f(x_1,x_2,\dots,x_n)$ の偏微分が、

$$
\begin{aligned}
\Delta f&=f(x_1+\Delta x_1,\ x_2+\Delta x_2,\ \dots\ ,x_n+\Delta x_n)-f(x_1,x_2,\dots,x_n)\\
&\sim\sum_{k=1}^n\frac{\partial f}{\partial x_k}\Delta x_k
\end{aligned}
$$

であることと比べると解決する。汎関数 $S$ は $t_1$ から $t_2$ までのすべての時刻に対する $q_k(t)$ の値を引数とする超多変数関数であると見做せるのだ。それら個々の値の変化に、その値に対する微分値 $\frac{\delta S}{\delta q_k}$ を掛けて足したものが全体としての変化となるのだ。
であることと比べると解決する。汎関数 $S$ は $t_1$ から $t_2$ までのすべての時刻に対する $q_k(t)$ の値を引数とする超多変数関数であると見做せるのだ。それら個々の値の変化に、その値に対する微分値 $\frac{\delta S}{\delta q_k}$ を掛けて足したものが全体としての変化となる。

ということで、当然だが汎関数微分 $\frac{\delta S}{\delta q_k}$ は時刻の関数となる。

** 汎関数微分がゼロとなる [#ifcb74cb]
** 最小点においては汎関数微分がゼロとなる [#ifcb74cb]

$f(x)$ の極大点・極小点において $\frac{df}{dx}=0$ が成り立つのと同様に、最小作用を与える $q_k(t)$ に対して、

$$\frac{\delta S}{\delta q_k}=0$$

が成り立つ必要がある。

実際に汎関数微分を計算してみると、
実際に汎関数微分を計算してみるにあたり $q_k(t)$ を変化させると $\dot q_k(t)$ も変化することを加味し、さらに $\Delta q_k(t_1)=\Delta q_k(t_2)=0$ を用いた部分積分を適用すると、

$$\begin{aligned}
\Delta S
&=\int_{t_1}^{t_2}\bigg[\frac{\partial L(\dots,q_k(t),\dot q_k(t),\dots)}{\partial q_k(t)}\Delta q_k(t)+\frac{\partial L(\dots,q_k(t),\dot q_k(t),\dots)}{\partial \dot q_k(t)}\Delta \dot q_k(t)\bigg]\,dt\\
&=\bigg[\frac{\partial L(\dots,q_k(t),\dot q_k(t),\dots)}{\partial \dot q_k(t)}\Delta  q_k(t)\bigg]_{t_1}^{t_2}+\\
&\phantom{=}\int_{t_1}^{t_2}\bigg[\frac{\partial L(\dots,q_k(t),\dot q_k(t),\dots)}{\partial q_k(t)}\Delta q_k(t)-\frac{d}{dt}\bigg\{\frac{\partial L(\dots,q_k(t),\dot q_k(t),\dots)}{\partial \dot q_k(t)}\bigg\}\Delta q_k(t)\bigg]\,dt\\
&=\int_{t_1}^{t_2}\underbrace{\bigg[\frac{\partial L(\dots,q_k(t),\dot q_k(t),\dots)}{\partial q_k(t)}-\frac{d}{dt}\bigg\{\frac{\partial L(\dots,q_k(t),\dot q_k(t),\dots)}{\partial \dot q_k(t)}\bigg\}\bigg]}_{=\,\frac{\delta S}{\delta q_k}}\Delta q_k(t)\,dt\\
\end{aligned}$$

任意の $\Delta q_k(t)$ に対してこれがゼロとなるには、任意の時刻に対して
を得る。ここで汎関数微分をゼロとすると以下を得る。

$$
-\frac{\delta S}{\delta q_k}=\frac{d}{dt}\bigg\{\frac{\partial L}{\partial \dot q_k}\bigg\}-\frac{\partial L}{\partial q_k}=0
{}-\frac{\delta S}{\delta q_k}=\frac{d}{dt}\bigg\{\frac{\partial L}{\partial \dot q_k}\bigg\}-\frac{\partial L}{\partial q_k}=0
$$

が成り立つ必要がある。
この方程式はラグランジュの運動方程式と呼ばれ、任意の $k$ に対してこの式が成り立つことが「停留作用」の法則の微分形を与える。

この方程式はラグランジュの運動方程式と呼ばれ、任意の $k$ に対してこの式が成り立つことが「停留作用」の法則の微分系を与える。(この式は最小となることを保証しない)
ラグランジュの運動方程式自体は作用が最小となることを保証しない。作用が停留値を取ることのみを与える。しかし、求まった軌道の近傍で小刻みに振動するような軌道を考えることで、ポテンシャルエネルギーをほとんど変えないまま運動エネルギーを増加させることはいつでも可能である。したがって、求まる停留値は常に最小の作用を与えることになる。

** ニュートン方程式との親和性 [#a6468f36]
** ニュートン方程式と矛盾しないこと [#a6468f36]

ポテンシャルエネルギー $U(x,y,z)$ 中で運動する質量 $m$ を持つ質点の運動は、ニュートン方程式

$$
\begin{cases}
m\ddot x=-\frac{\partial}{\partial x} U\\
m\ddot y=-\frac{\partial}{\partial y} U\\
m\ddot z=-\frac{\partial}{\partial z} U\\
\end{cases}
$$

で記述される。
で記述される。(力は $\bm f=-\bm \nabla U$ で与えられることを思い出せ)

一方、この系の運動エネルギーは $T=\frac{1}{2}m(\dot x^2+\dot y^2+\dot z^2)$ であるから、

$$
L=T-U=\frac{1}{2}m(\dot x^2+\dot y^2+\dot z^2)-U(x,y,z)
$$

である。

ラグランジュの運動方程式は、

$$
\begin{cases}
\displaystyle\frac{d}{dt}\bigg\{\frac{\partial L}{\partial \dot x}\bigg\}=\frac{\partial L}{\partial x}
\ \ \to\ \ \frac{d}{dt}m\dot x=\frac{\partial}{\partial x}U\\
\ \ \to\ \ \frac{d}{dt}(m\dot x)=-\frac{\partial}{\partial x}U\\
\displaystyle\frac{d}{dt}\bigg\{\frac{\partial L}{\partial \dot y}\bigg\}=\frac{\partial L}{\partial y}
\ \ \to\ \ \frac{d}{dt}m\dot y=\frac{\partial}{\partial y}U\\
\ \ \to\ \ \frac{d}{dt}(m\dot y)=-\frac{\partial}{\partial y}U\\
\displaystyle\frac{d}{dt}\bigg\{\frac{\partial L}{\partial \dot z}\bigg\}=\frac{\partial L}{\partial z}
\ \ \to\ \ \frac{d}{dt}m\dot z=\frac{\partial}{\partial z}U
\ \ \to\ \ \frac{d}{dt}(m\dot z)=-\frac{\partial}{\partial z}U
\end{cases}
$$

となって、確かにニュートン方程式と一致する。

実はラグランジアンや作用を適切に定義することにより、
ニュートン方程式だけでなくマクスウェル方程式や一般相対論などについても
最小作用の形で定式化できることが知られている。


* 例題 [#qd0685db]

$x$ 軸上に限り摩擦無く自由に動ける点電荷 $q$ (質量 $m$) が $t=0$ において $(-1,0)$ において静止している。$t>0$ においてこの点電荷が
ラグランジュ方程式がニュートン方程式と同一ならば、なぜ新たにラグランジュ方程式を考える必要があるのだろう。その一端を見るために次の例題を考える。

** $x$ 軸上の点電荷 [#ic5a4aaf]

$x$ 軸上に限り摩擦なく自由に動ける点電荷 $q$ (質量 $m$) が $t=0$ において $(-1,0)$ において静止している。$t>0$ においてこの点電荷が
$(0,1)$ に固定された点電荷 $-q$ からのクーロン力と、運動を $x$ 軸上に限るための拘束力のみを受けて運動するとして、点電荷の $x$ 座標の時間変化を表す運動方程式を求めよ。ただし系の誘電率を $\epsilon_0$ とする。

普通にやるなら、
これまでと同様に普通にニュートン方程式を立てるなら、

電荷間の引力 $-\frac{1}{4\pi\epsilon_0}\frac{e^2}{x^2+1}$

$x$ 軸に沿った成分は $-\frac{1}{4\pi\epsilon_0}\frac{e^2}{x^2+1}\frac{x}{\sqrt{x^2+1}}$

したがって、

$$
m\ddot x=-\frac{1}{4\pi\epsilon_0}\frac{e^2}{x^2+1}\frac{x}{\sqrt{x^2+1}}
$$

一方、ラグランジアンを使うと、

運動エネルギー $T=\frac{1}{2}m\dot x^2$

ポテンシャルエネルギー $U=\frac{1}{4\pi\epsilon_0}\frac{e^2}{\sqrt{x^2+1}}$

ラグランジアン $L=T-U$

$$
L=\frac{1}{2}m\dot x^2-\frac{1}{4\pi\epsilon_0}\frac{e^2}{\sqrt{x^2+1}}
$$

運動方程式
ラグランジュの運動方程式

$$
\frac{d}{dt}\left(\frac{\partial L}{\partial \dot x}\right)-\frac{\partial L}{\partial x}=0
$$

$$
m\ddot x=-\frac{e^2}{4\pi\epsilon_0}\frac{x}{(x^2+1)^{3/2}}
$$

として求められる。
として同じ式が得られる。

** 振り子の運動 [#sebbcf9b]

長さ $r$、重りの質量 $m$ の振り子について、その振れ角を $\theta$ とすると、
重りの速度は $r\dot\theta$、高さは $r-r\cos\theta$ と表せるから、
重力加速度を $g$ として、

運動エネルギー $T=\frac12 mr^2\dot\theta^2$

ポテンシャルエネルギー $U=mgr(1-\cos\theta)$

ラグランジアン $L=\frac12 mr^2\dot\theta^2-mgr(1-\cos\theta)$

$\theta$ に対する運動方程式は、

$$
\frac{d}{dt}\left\{\frac{\partial L}{\partial\dot\theta}\right\}=\frac{\partial L}{\partial\theta}
$$

$$
mr^2\ddot\theta=-mgr\sin\theta
$$

すなわち、

$$
\ddot\theta=-\frac{g}{r}\sin\theta
$$

を得る。

** 運動をラグランジアン形式で記述することのメリット [#z9cb4f3d]

ラグランジアンを用いた方法では力の働く方向などを考える必要がなく、
ラグランジアンさえ求めてしまえばそれを形式的に微分するのみで
運動方程式が得られることに注目したい。

また、運動を記述するのが $x,y,z$ 座標であろうが、$r,\theta,\phi$ であろうが、
基本方程式が同じ形で記述できること(運動方程式が座標変換に対して共変である、という)
も非常に重要である。

* ラグランジュの運動方程式の共変性 [#kf999ae0]

実際、座標系 $q_1,q_2,\dots,q_n$ から別の座標系 $Q_1,Q_2,\dots,Q_n$ への変数変換が、

$$
Q_i=Q_i(q_1,q_2,\dots,q_n)\ \ \ \ \ (i=1,2,\dots,n)
$$

で与えられるとして、ラグランジュの運動方程式の共変性を導いておこう。

この変換には逆変換が存在して、

$$
q_i=q_i(Q_1,Q_2,\dots,Q_n)\ \ \ \ \ (i=1,2,\dots,n)
$$

とする。

この座標変換は $x,y,z$ から $r,\theta,\phi$ のような幾何学的な座標変換であっても良いし、
2つの粒子の座標を重心座標と相対座標とに分けたり、あるいは
バネで連結された多数の質点の個々の位置座標から基準振動の振幅と位相へ変換するなど、どんな変換でも構わない。

座標 $q_1,q_2,\dots,q_n$ を決めれば座標 $Q_1,Q_2,\dots,Q_n$ を求められ、
逆に座標 $Q_1,Q_2,\dots,Q_n$ を決めれば座標 $q_1,q_2,\dots,q_n$ を求められる、
ということだけが座標変換の条件である。

このとき、すべての $i=1,2,\dots,n$ に対して、

$$
\frac{d}{dt}\left\{\frac{\partial L}{\partial \dot q_i}\right\}-\frac{\partial L}{\partial q_i}=0
$$

が成り立つことを仮定して、すべての $i=1,2,\dots,n$ に対して、

$$
\frac{d}{dt}\left\{\frac{\partial L}{\partial \dot Q_i}\right\}-\frac{\partial L}{\partial Q_i}=0
$$

が成り立つことを以下のように導ける。

$$
\dot q_i=\sum_{j=1}^n \frac{\partial q_i}{\partial Q_j}\dot Q_j
$$

であるから、

$$
\frac{\partial L}{\partial Q_i}=\sum_{j=1}^n\frac{\partial q_j}{\partial Q_i}\frac{\partial L}{\partial q_j}
$$

$$
\begin{aligned}
\frac{\partial L}{\partial\dot Q_i}
&=\sum_{j=1}^n\frac{\partial\dot q_j}{\partial\dot Q_i}\frac{\partial L}{\partial\dot q_j}\\
&=\sum_{j=1}^n\frac{\partial q_j}{\partial Q_i}\frac{\partial L}{\partial\dot q_j}\\
\end{aligned}
$$

代入すると、

$$
\begin{aligned}
\frac{d}{dt}\left\{\frac{\partial L}{\partial \dot Q_i}\right\}-\frac{\partial L}{\partial Q_i}
&=\sum_{j=1}^n\frac{\partial q_j}{\partial Q_i}\underbrace{\left[\frac{d}{dt}\left\{\frac{\partial L}{\partial \dot q_i}\right\}-\frac{\partial L}{\partial q_i}\right]}_{=\,0}=0\\
\end{aligned}
$$

を得る。

* ラグランジアンの不定性 [#mcce419c]

座標系 $q_1,q_2,\dots,q_n$ から別の座標系 $Q_1,Q_2,\dots,Q_n$ への変数変換が、

$$
Q_i=Q_i(q_1,q_2,\dots,q_n)\ \ \ \ \ (i=1,2,\dots,n)
$$

で与えられるとしよう。* ラグランジアンの任意性

ある系のラグランジアン $L$ に、ある関数 $F$ の時間微分 $\frac{dF}{dt}$ を加えても
運動方程式は変化しない。なぜなら、

$$
L'=L+\frac{dF}{dt}
$$

に対して、

$$
S=\int_{t_1}^{t_2}L'\ dt=\int_{t_1}^{t_2}\left(L+\frac{dF}{dt}\right)\ dt=\int_{t_1}^{t_2}L\ dt+\underbrace{F(t_2)-F(t_1)}_\text{定数}
$$

となって、$q_k(t_1),q_k(t_2)$ が固定されている限り $F(t_2)-F(t_1)$ は定数であり、
$S$ の $q_k$ による汎関数微分に影響を与えないためである。

したがって、ある力学系のラグランジアンを決定する際には $F$ の分だけの不定性が残されていることになる。

* 質問・コメント [#r1984971]

#article_kcaptcha


Counter: 45033 (from 2010/06/03), today: 5, yesterday: 0